Wenn ein Goldstone-Boson eine Erregung ist, die sich zwischen entarteten Vakuen bewegt, wie bleiben dann Symmetrien gebrochen?

Beim spontanen Symmetriebrechen ändert das Bewegen um das kreisförmige Tal des mexikanischen Hutpotentials die potentielle Energie nicht. Diese Winkelanregungen werden Goldstone-Bosonen genannt. Aber impliziert die Änderung des Winkels nicht, dass sich das System von einem Vakuum zum anderen bewegt, weil verschiedene Punkte auf dem kreisförmigen Tal entartetes Vakuum darstellen?

Wenn Goldstone-Anregungen so sind, wie bleibt die Symmetrie gebrochen? Die Goldstone-Anregung wird per Definition (weil sie Variationen in den Koordinaten auf dem kreisförmigen Tal darstellen) das System dann von einem Vakuum in ein anderes bringen. Dies geschieht jedoch nicht.

Ich denke, Sie fragen sich, ob sich der Zustand im Tal bewegt oder ob er fest bleibt ("bleibt in derselben Ausrichtung kaputt"), und wenn er fest bleibt, was hält ihn dort? ... Ich bin mir nicht sicher, also hinterlasse ich dies einfach als Kommentar. Aber ich denke, man kann an jedem Punkt im Raum einen anderen Wert des Feldes (Position im Tal) haben, und es spielt keine Rolle, welcher es ist, weil man das Feld nicht direkt messen kann. Was zählt, ist die Form des Potentials - die Position des Minimums gibt Ihnen den Higgs-VEV, die Krümmung (Schwingungen an den Wänden hinauf) erzeugt die Masse der Eichbosonen.
Danke. Es ist wahr, dass das Klettern auf eine potenzielle Wand Higgs Masse verleiht, weil Masse in der Feldtheorie mit dem Koeffizienten des Quadrats des Feldwerts in der Lagrangian identifiziert wird. Aber meine Frage bezog sich auf Goldstone-Bosonen, nicht auf Higgs. Lassen Sie es mich in präzise Worte fassen. Wie können Sie sich im kreisförmigen Tal bewegen (Goldstein-Anregungen), wenn die Symmetrie spontan gebrochen ist (was bedeutet, dass Sie an einem Punkt auf dem Kreis fixiert bleiben sollen)?
Was ich sagen wollte, war, dass die Symmetriebrechung nur impliziert, dass Sie sich irgendwo im Kreis befinden, nicht, dass Sie sich an einem bestimmten Fixpunkt befinden. Ich weiß nicht, wie Sie sich im Tal bewegen, oder ob die "Winkelposition" eine physikalische Bedeutung hat, aber ich würde es auch gerne wissen.
Ich denke, das ist eine gute Frage, ich bin mir nicht sicher, was die Verwirrung ist. Die Frage ist, bleibt das System nach einer spontanen Symmetriebrechung in einem bestimmten Vakuum oder bewegt es sich. Darüber hinaus hat dies irgendwelche Auswirkungen.
die vacua sind disjunkt, θ | ϕ = δ ( ϕ θ ) , sobald du also in einem Vakuum bist, bleibst du dort. auch superselektionsregeln sind meiner meinung nach relevant, weil sie die präparierung einer superposition von vacua verbieten, zb | 0 = | ϕ + | θ , also können Sie die Symmetrie mit eg nicht bewahren | 0 = D θ | θ
Bitte lassen Sie Posts nicht wie Revisionshistorien aussehen . Das heißt, fügen Sie bitte keine Markierungen wie „Bearbeiten:“ oder „Aktualisieren:“ hinzu, wenn Sie eine Bearbeitung vornehmen.
Die Frage selbst ist mir genauso unklar wie den anderen Kommentatoren. Ich vermute, dass das Problem Ihr Verständnis davon ist, was ein Goldstone-Boson ist. Wo genau haben Sie beim Beweis des Satzes von Goldstone den Eindruck gewonnen, dass die Goldstone-Bosonen wirklich eine "Bewegung" entlang des Vakuumverteilers induzieren? Es ist manchmal ein heuristisches Bild, aber da die verschiedenen Vakuen keine Überlappung haben, können solche störenden Erregungen nicht zwischen verschiedenen Vakuen vermitteln.
@ACuriousMind Ich denke, es ist das heuristische Bild, das mich verwirrt hat. Bücher sagen oft, dass Goldstone-Bosonen masselos sind, weil "die Bewegung auf dem kreisförmigen Tal keine Energie kostet", weil es keine Änderung der Krümmung gibt, dh 2 v ( ϕ ) ϕ 2 = 0 für Goldstone-Feld ϕ . ϕ ist auch die Koordinate auf dem kreisförmigen Tal des mexikanischen Hutpotenzials.
@SRS Ich habe meine Antwort um weitere Details und eine Diskussion erweitert. Bitte guck dir das an.

Antworten (4)

Das ist eine gute Frage. Ich glaube, Sie wenden eine falsche Analogie mit einem Beispiel an, das oft verwendet wird, um zuerst den Begriff SSB einzuführen: ein einzelnes nichtrelavistisches Teilchen in einem Doppeltopf mit einer Potentialbarriere, die die beiden Minima trennt. Solange die Barriere zwischen den Minima endlich ist, kann das Teilchen sie durchtunneln und beide Minima bewohnen, sodass die Reflexionssymmetrie ungebrochen ist. Aber wenn die Barrierenhöhe formal ins Unendliche gebracht wird, bleibt das Teilchen in dem einen oder anderen Minimum „stecken“, wodurch die Symmetrie gebrochen wird. Sie denken, dass es bei einem Potenzial vom Typ Mexikanerhut keine Potenzialbarriere gibt, sodass das System frei sein sollte, zu allen Minima zu tunneln und die Symmetrie wiederherzustellen.

Aber eine unendlich hohe Potentialbarriere ist eine ziemlich künstliche Vorstellung. Es ist wirklich eine schematische Darstellung einer realistischeren Situation, in der Sie statt eines einzelnen Teilchens ein räumlich ausgedehntes Feld (oder ein Vielteilchensystem auf einem riesigen Gitter) haben, das über ein großes Volumen definiert ist v . Dann repräsentieren die beiden Minima im Ein-Teilchen-Bild wirklich zwei unterschiedliche Feldkonfigurationen mit gleicher Gesamtenergie. Da der Übergang von einer Konfiguration zur anderen eine Änderung des Feldwerts an jedem einzelnen Punkt im Raum (oder Raumzeit) erfordert, erfordert dies eine enorme Energie, die proportional zum gesamten Systemvolumen ist v . Die „Barrierenhöhe“ im Ein-Teilchen-Bild entspricht also wirklich dem Volumen v des Systems im feldtheoretischen Bild. Nur in der formalen Grenze des unendlichen Volumens ("thermodynamische") ist die Symmetrie wirklich gebrochen.

Verschieben Sie nun die Feldkonfiguration von beispielsweise θ ( X ) 0 Zu θ ( X ) δ für einen winzigen Winkel δ erfordert nur eine winzige Energiedichte proportional zu δ (in den entsprechenden Einheiten), sondern die Gesamtenergie δ × v kann noch sehr groß werden. Es ist eine Subtilität der Ordnung der Grenzen: für jede Schicht δ im Feld Wert, egal wie klein, wir können uns ein System so groß vorstellen (grob gesagt viel größer als 1 / δ ), dass das Verschieben des gesamten Systems um diesen Betrag eine beliebig große Energiemenge erfordert. So erhält man im Ein-Teilchen-Bild trotz der potentiellen Energiedichte des Feldes immer noch eine "unendlich hohe Potentialbarriere zum Übertunneln". v ( φ ) hat überhaupt keine Barriere.

(Um die Dinge expliziter quantenmechanisch zu machen, betrachten Sie das Quanten-Heisenberg-Modell auf einem Gitter. Wenn | ψ Und | ψ ' sind zwei individuelle Spin- 1 / 2 s auf der Blochkugel um einen kleinen Winkel gedreht δ , dann das innere Produkt ψ | ψ ' = cos δ 1 ( 1 / 2 ) δ 2 ist ziemlich groß, also ein Spin- 1 / 2 könnte leicht zwischen den beiden Staaten tunneln. Aber wenn wir zwei riesige Systeme von betrachten N 1 ausgerichtete Spins | Ψ = ich = 1 N | ψ ich Und | Ψ ' = ich = 1 N | ψ ' ich , dann die Tunnelamplitude Ψ | Ψ ' = cos ( δ ) N zwischen den beiden Systemen ist winzig, daher ist es sehr schwierig, einen Zustand in den anderen umzukehren.)

(Um all dies in die Antwort von TwoBs einzubinden: In der formalen Grenze des unendlichen Volumens wird die Fourier- Reihendarstellung des Felds zu einer kontinuierlichen Fourier- Transformation , die durch einen kontinuierlichen Parameter indiziert wird k , und wir können über Taylor-Erweiterung der Energiedichte-Dispersionsbeziehung sprechen ϵ ( k ) um k = 0 . Für einen Goldstone-Modus haben wir ϵ 0 als k 0 , aber das ist nur eine Energiedichte - die Gesamtenergie E ( k ) = v × ϵ ( k ) ist immer noch riesig, also ist die "Energiebarriere" immer noch sehr hoch. Ein Goldstone-Modus müsste räumlich unendlich erweitert werden, um die Fernordnung wirklich zu zerstören und die Symmetrie wiederherzustellen, und ein so unendlich großer Goldstone-Modus ist immer noch zu energetisch kostspielig, um ihn zu erzeugen.)

kleiner Kommentar: Ich glaube nicht, dass sich das bewegt θ = 0 Zu θ = δ erfordert eine Energiedichte proportional zu δ da das die Energiedichte ist θ -unabhängig. Es sind alle Steigungen θ die für Goldstone-Bosonen von Bedeutung sind. Man könnte sich vielleicht einen sehr adiabatischen Prozess vorstellen, wo alle θ -Ableitungen sehr klein sind und sie schneller auf Null bringen als die Lautstärke erhöht wird, bin ich mir nicht sicher ... Aber ich stimme zu, dass der entscheidende Aspekt bei der Unterdrückung der Amplitude die unendliche Lautstärke ist, die sich mathematisch in den verschwindenden Amplituden im Soft Limit widerspiegelt.
@TwoBs Ja, ich denke, es hängt genau davon ab, wie Sie es tun. Wenn Sie winzige kleine Domänen einzeln verschieben, würden die Energiekosten für jede kleine Domäne wie folgt aussehen: (Oberfläche der Domänenwand) * ( δ / Domänenwandstärke), also δ mal ein geometrischer Faktor. Man könnte auch eine große Region auf sehr kontrollierte, gleichmäßige Weise spiegeln, um alle räumlichen Gradienten klein zu halten, aber natürlich ist eine so fein abgestimmte, nicht lokale Operation nicht sehr realistisch. Vielleicht haben Sie Recht damit, dass ein sehr adiabatischer Prozess die Energiekosten beliebig klein machen kann.
@tparker Ich habe hier eine verwandte Frage gestellt . physical.stackexchange.com/questions/437175/… irgendwelche Gedanken dazu?

Ich bin mir nicht sicher, ob ich die Frage ganz verstehe, aber ich werde es versuchen.

Ich denke, die Antwort auf Ihre Frage liegt im Nullzustand des Adlers. Tatsächlich würden die Goldstone-Bosonen (GB) nur in der Nullimpulsgrenze ein neues Minimum darstellen (andernfalls erhöht ihre kinetische Energie die Gesamtenergie und erzeugt Raumzeitgradienten, die für das Vakuum nicht vorhanden sind), was genau die Grenze ist für die die Amplituden von GB verschwinden. Es findet also kein nicht-trivialer Übergang statt.

Zusätzliche Bearbeitungen nach zusätzlichen Gedanken Mein Gefühl, dass meine obige Antwort sinnvoll und richtig ist, wird verstärkt, wenn ich darüber nachdenke, wie Sie tatsächlich von einem Vakuum zum anderen wechseln würden, nämlich indem Sie mit dem Exponential des (möglichen *) Ladungsoperators handeln Q = D 3 X J 0 = lim P 0 J ^ 0 ( P ) . Außer dass für eine spontan gebrochene kontinuierliche globale Symmetrie der Strom linear im GB-Feld beginnt, J μ = F μ π + , so dass Q | 0 versetzt Sie nicht in ein anderes Vakuum, sondern in einen kohärenten Zustand von Null-Impuls-GBs, beginnend mit einem einzelnen weichen GB aus einem Teilchen: Q | 0 = lim P 0 | π ( P ) + , für die wiederum die Amplituden verschwinden. Dies wird in Abschnitt 4.1 dieses schönen Artikels https://arxiv.org/abs/0808.1446 gut erklärt (und tatsächlich werden die weichen Theoreme sogar davon abgeleitet) .

* Bonuskommentar: Man könnte sich Sorgen über das Fabri-Picasso-Theorem machen (siehe z. B. https://en.wikipedia.org/wiki/Goldstone_boson ), das uns sagt, dass die Ladung streng genommen nicht wirklich existiert (obwohl ihr Kommutator macht immer). Aber diese Aussage ist ein Overkill, da es einfach die Aussage ist, dass die Ein-Teilchen-Zustände bestimmter Impulse, wie z | π ( P 0 ) die durch die Ladung erzeugt wird, haben unendliche Norm, dh π ( P ) | π ( k ) = ( 2 π ) 3 2 | k | δ 3 ( k P ) . Übrigens ist diese IR-Abweichung von der Norm des Staates z k P = 0 ist proportional zum Volumen v , Kontakt aufnehmen mit der Antwort von @tparker . Die Moral dieser Geschichte ist: Die Ladung existiert streng genommen nicht, sondern nur, weil man Ein-Teilchen-Zustände weicher Impulse erzeugt. Es ist durchaus sinnvoll, Soft Limits in Betracht zu ziehen, indem man das Limit sorgfältig betrachtet, wiederum wie in der oben zitierten Referenz.

Die Aussage, dass die Symmetrie G spontan gebrochen wird, bedeutet , dass wir durch Einwirkung von G auf die Vakuumkonfiguration eine isomorphe, aber unterschiedliche Konfiguration erhalten. Damit die Symmetrie ungebrochen ist, müssten die Transformationen in G die Vakuumkonfiguration auf dieselbe abbilden, nicht nur auf eine isomorphe.

Wenn Sie den Buchstaben R entlang der vertikalen Achse spiegeln, erhalten Sie ß. Dieses "ya" ist isomorph, aber es ist anders, also ist R nicht links-rechts-symmetrisch; die Symmetrie ist gebrochen; Es ist niemals möglich, dass eine Symmetrie ein Objekt erzeugt, das auch nur anders aussieht (nicht isomorph ist). Es ist immer isomorph; die Frage ist, ob es identisch ist . Der Buchstabe H wird wieder auf H abgebildet, also ist H links-rechts-symmetrisch.

Dass die Goldstone-Bosonen nichttriviale Anregungen sind, beweist, dass die Wirkung von G nichttrivial ist, sodass das Vakuum unter G nicht symmetrisch ist.

Ja. Nachdem die Symmetrie (z. B. U(1)) spontan gebrochen ist, wählt das System zufällig eine bestimmte Lösung, eine von unendlich vielen solcher Lösungen. Es ist auch wahr, dass die Lösungen nach dem Aufbrechen der Symmetrie nicht mehr die Symmetrie der Lagrange-Funktion aufweisen und die Wirkung eines U(1)-Gruppenelements eine Lösung auf eine andere abbildet. Aber das war nicht meine Verwirrung...
Toll, dann habe ich keine Ahnung, was Ihre Frage möglicherweise sein könnte.
Was meinen Sie, wenn Sie im Wesentlichen sagen, dass die entarteten Vakuen isomorph zueinander sind? Ich kenne Isomorphismen zwischen zwei Gruppen. @LubosMotl
Lieber @SRS - Isomorphismus ist nicht nur für Gruppen definiert, sondern auch für Ringe, Felder, Lie-Algebren, was auch immer - und für Objekte. Der allgemeine Punkt ist, dass es eine Karte – den Isomorphismus – von einem zum anderen gibt, die einfach ist, auf und die alle Strukturen auf dem mathematischen oder physikalischen Objekt respektiert. Sie können also die Hamiltonianer, Operatoren, zuordnen, sie haben die Operatoren, die nur Übersetzungen voneinander sind usw. Aber sie sind nicht "gleich", Sie können sie nicht wirklich Element für Element in derselben Menge / demselben Raum identifizieren. Sie haben nur die gleiche Form - iso morph ;-).

Ich bin mir nicht sicher, ob ich die Frage verstehe. Die Symmetrie ist gebrochen, weil du im Tal bist. Daran ändert, wie Sie richtig sagen, die Bewegung im Tal nichts.